电子线路 非线性部分 (参考)_khdaw

电子线路 非线性部分 (参考)

1-2 一功率管,它的最大输出功率是否仅受其极限参数限制?为什么?

解:否。还受功率管工作状态的影响,在极限参数中,P CM 还受功率管所处环境温度、散热条件等影响。

1-3 一功率放大器要求输出功率P 。= 1000 W,当集电极效率ηC 由40%提高到70‰时,试问直流电源提供的直流功率P D 和功率管耗散功率P C 各减小多少?

解:

当ηC1 = 40% 时,P D1 = P o /ηC = 2500 W,P C1 = P D1 − P o =1500 W

当ηC2 = 70% 时,P D2 = P o /ηC =1428.57 W,P C2 = P D2 − P o = 428.57 W 可见,随着效率升高,P D 下降,(P D1 − PD2) = 1071.43 W

P C 下降,(P C1 − PC2) = 1071.43 W

1-6 如图所示为低频功率晶体管3DD325的输出特性曲线,由它接成的放大器如图

试求下列条件下的P L 、P D 、ηC (运用图解法):(1)R L = 10Ω,1-2-1(a )所示,已知V CC = 5 V,

(3)R L = 5Ω,Q 点Q 点在负载线中点,充分激励;(2)R L = 5 Ω,I BQ 同(1)值,I cm = I CQ ;

在负载线中点,激励同(1)值;(4)R L = 5 Ω,Q 点在负载线中点,充分激励。

k h

d a

w w

1

所以P L =V cm I cm =264mW ,P D = V CC ICQ1 =

2

1.1 W,ηC = PL / PD = 24%

(2) 当 R L = 5 Ω 时,由V CE = V CC − I C R L 作负载线,I BQ 同(1)值,即I BQ2 = 2.4mA,得Q 2点,V CEQ2 = 3.8V,I CQ2 = 260mA

这时,V cm = V CC −V CEQ2 = 1.2 V,I cm = I CQ2 = 260 mA

1

所以 P L =V cm I cm =156mW ,P D = V CC ICQ2 = 1.3 W,ηC = PL / PD = 12%

2

w .

解:(1) R L = 10 Ω 时,作负载线(由V CE = V CC − I C R L ) ,取Q 在放大区负载线中点,充分

I CQ1 = 220mA,I BQ1 激励,由图得V CEQ1 = 2.6V,

= I bm

= 2.4mA

因为V cm = V CEQ1−V CE(sat) = (2.6 − 0.2) V = 2.4 V,I cm = I CQ1 = 220 mA

(3) 当 R L = 5 Ω,Q 在放大区内的中点,激励同(1),

由图Q 3点,V CEQ3 = 2.75V,I CQ3= 460mA,I BQ3 = 4.6mA, I bm = 2.4mA 相应的v CEmin = 1.55V,i Cmax = 700mA。

第 1 页 共 27 页

w . c o

m

因为V cm = V CEQ3 − v CEmin = 1.2 V,I cm = iCmax − I CQ3 = 240 mA

1

所以P L =V cm I cm =144mW ,P D = V CC ICQ3 = 2.3 W,ηC = PL / PD = 6.26%

2

(4) 当 R L = 5 Ω,充分激励时,I cm = I CQ3 = 460 mA,V cm = V CC −V CEQ3 = 2.25 V 1

所以 P L =V cm I cm =517. 5mW ,P D = V CC ICQ3 = 2.3 W,ηC = PL / PD = 22.5%

2

设V CE(sat) 1-7 如图所示为三种甲类功率放大器的输出电路,采用相同的功率管及V CC 值。

= 0,I CEO = 0,变压器是理想无耗的,试在同一输出特性曲线上作出各电路的交、直流负载

1 直流负载线方程 v CE = V CC − i C R C ,负载线CD ,当i C = I CQ 时,V CEQ = V CC 解:(1) ○

− I CQ R C 。

′ I cm = I CQ,V cm = V CEQ = I

cm R L

′= VCC − 2V cm = V CC − 2I cm R L

解得

w w

w .

1

V cm =V CC ,I cm

3

P L max(a ) =

k h

1V CC 3 =′R L

11

×V CC 23

代入V CEQ 方程中 V cm = V CC − Icm R C = V CC − ICQ R C

2 交流负载线中点过Q ,斜率为(−1/R L ′) ,R L ′=R L //R C =○

1

V CC 2

1V CC ×=′′R L 18R L

d a

P L max(a )

P D

=1

6

P D =V CC I CQ =V CC I cm

所以 ηC =

(2) 交流负载相同,均为CF ,为获最大输出功率,Q 处于交流负载线的中点,故 V cm = V CEQ = VCC /2,I cm =I CQ =

V CC

2R L

第 2 页 共 27 页

w . c o

1V CC

=

′3R L

2

1

R C ,根据交流负载线 AB 2

m

线,并求这三种放大器的最大输出功率之比P L max(a ) :P L max(b ) :P L max(c ) 。

所以 P L max(b )

22

V CC 11V CC

=V cm I cm =;P D =V CC I CQ =

28R L 2R L

ηC(b)=

P L max(b )

P D

=

1

4

′) 的直线MN ,(3) 因为直流负载电阻为零,故直流负载线为CG ,交流负载线斜率为(−1/R L

当Q C 处于中点时,得

V cm = V CEQ = VCC ,I cm =I CQ =

所以P L max(a ) :P L max(b ) :P L max(c ) =

111

::=4:9:36 1882

ηC(a):ηC(b):ηC(c)=

′、I bm 不变,将V CC 增加一倍,求不变,将I CQ 增加一倍,求P L 值;(3)保持(1)中I CQ 、R L

P L 值;(4)在(3)条件中,将I bm 增加一倍,试分析工作状态。

w w

w .

′= 50 Ω,负载解:(1) 因为V CC = 15 V,R L

第 3 页 共 27 页

k h

′ = 50 Ω,(1)V CC = 15 V,R L 在负载匹配时,求相应的n 、P Lmax 、ηC ;(2)保持(1)中V CC .I bm

d a

1-8 如图(a )所示为变压器耦合甲类功率放大电路,图(b )所示为功率管的理想化输出特性曲线。已知R L = 8 Ω,设变压器是理想的,R E 上的直流压降可忽略,试运用图解法:

111

::=2:3:6 642

w . c o

P L max(c )

22

P L max(c ) 1V CC 11V CC

所以ηC(c)=,P D =V CC I CQ =,= =V cm I cm =

′′22R L R L 2P D

m

V CC

′R L

匹配时,I CQ1=I cm =

V CC

=0. 3A ′R L

由此得知Q 1的坐标为Q 1(15V,0.3A) ,Q 1点处于交流负载线AB 的中点,其在坐标轴上的

截距为A (32 V,0) ,B (0,0.6A) 。由图可见

I cm = I CQ1=0.3A,V cm = V CC = 15 V 1

此时,P Lmax =V cm I cm =2. 25W ,

2

P D =V CC I CQ =4. 5W

′> 50 Ω,交流负载线以Q 2为中心逆时针转动,但由于激励不变,输出将出现饱和当R L

失真。

(3) V CC = 30 V,交流负载线平移到EF ,静态工作点为Q 3,因为I bm 不变,所以V cm 不变,I cm 不变,因此P L 不变,P L = 2.25 W,但V CC = 30 V,所以

P D = V CC × I CQ = 9 W ηC = P L / PD = 25%

(4) I bm = 6 mA,以Q 3点为静态工作点,出现截止失真。

1-9 单管甲类变压器耦合和乙类变压器耦合推挽功率放大器采用相同的功率管

′等于匹配值,3DD303、相同的电源电压V CC 和负载R L ,且甲类放大器的R L 设V CE(sat) = 0, I CEO

= 0,R E 忽略不计。(1)已知V CC = 30 V,放大器的i Cmax = 2 A,R L = 8 Ω,输入充分激励,试

w

′、n ;作交流负载线,并比较两放大器的P omax 、 P Cmax 、ηC 、R L (2)功率管的极限参数P CM =

30 W,I CM = 3 A,V (BR)CEO= 60 V,试求充分利用功率管时两放大器的最大输出功率P omax 。

解:(1) 见表

第 4 页 共 27 页

w .

k h

d a

′↓ → P L ↓ ηC ↓ I cm 不变,因而R L

(I bm 不变,I cm 不变,V cm 不变)

但 P D = V CC × I CQ = 9 W ηC = P Lmax / PD = 25%

2当R L ′改变时,且R L ′

1当R L ′不变时,因为I CQ 增加一倍,因此,R L ′已不是匹配值,其交流负载线平行移动,○

为一条过Q 2点的直线EF

′不变,斜率不变,I CQ 增加,Q 点升高) (R L

′都不变,其P Lmax 亦不变,为2.25 W 此时,由于V CC 、I bm 、R L

w .

c o

′是否变化没说明,故分两种情况讨论 (2) R L

m

ηC =

P L max 2. 25

==50%,n =P D 4. 5R L 50

==2. 5 R L 8

w w

w .

解:T 4、R 2、R 3组成具有直流电压并联负反馈的恒压源,给T 1、T 2互补管提供克服交越失真的直流正偏压。

(1) P L =

12

I cm R L =16W 2

(2) Q P D =(3) ξ=

V cm

=0. 8,∴ηC =(π/4) ×ξ=62. 83%

V CC /2

第 5 页 共 27 页

k h

(V CC I cm ) P −P L

=25. 47W ,=4. 74W ∴P C =D

π2

d a

1-14 如图所示为两级功放电路,其中,T l 、T 2工作于乙类,试指出T 4、R 2、R 3的作用。

,讨计算:(1)当R L = 8 Ω时的当输人端加上激励信号时产生的负载电流为i L = 2sinωt (A )

(2)每管的管耗P C ;(3)输出级的效率ηC 。设R 5、R 6电阻不计。 输出功率P L ;

w

(1)互补功率管为复合管;(2)1-16 试按下列要求画单电源互补推挽功率放大器电路:

(4)采用二极管偏置电路。 推动级采用自举电路;(3)引入末级过流保护电路;

解:按要求画出的单电源互补推挽功率放大器电路如图所示。图中T 1为推动级,T 2、T 3、T 4、T 5为准互补推挽功率级,D 1、D 2为末级偏置电路,T 6、T 7为过流保护电路,C 2为自举电容。

w w

解:(1) T 1、T 2和T 3、T 4为复合管组成差分放大器,作为推动级。T 5、T 6为镜像电流源,

T 7~ T10——准互补功放电路,D 1~ D3——为功率级提供正向偏置;R 5、作为差放级有源负载。

R 6——电压串联负反馈,改善电路性能。 (2) 通过R 1的电流I EE =

w .

k h

V EE −2V BE(on)

R 1

, I CQ5 = ICQ6 = 0.5 IEE ,k f v =

各管V BE(on)相等,设各管基极电流不计,求I CQ5(I CQ6) 及k f v 表达式。

1-17 两级功放原理电路如图所示。试:(1)简述电路工作原理;(2)已知V CC = EE ,

d a

2-1 为什么谐振功率放大器能工作于丙类,而电阻性负载功率放大器不能工作于丙类?

第 6 页 共 27 页

w .

c o

R 5v f

= v o R 5+R 6

m

解:因为谐振功放的输出负载为并联谐振回路,该回路具有选频特性,可从输出的余弦脉冲电流中选出基波分量,并在并联谐振回路上形成不失真的基波余弦电压,而电阻性输出负载不具备上述功能。

乙类有何优点?为什么?丙类工作的放大器适宜于2-2 放大器工作于丙类比工作于甲、

放大哪些信号?

解:(1)丙类工作,管子导通时间短,瞬时功耗小,效率高。

(2) 丙类工作的放大器输出负载为并联谐振回路,具有选频滤波特性,保证了输出信号的不失真。

为此,丙类放大器只适宜于放大载波信号和高频窄带信号。

2-4 试证如图所示丁类谐振功率放大器的输出功率P o =

效率ηC =和ηC 值。

V CC −2V CE (sat )

V CC

。已知V CC = 18 V,V CE(sat) = 0.5 V,R L = 50 Ω,试求放大器的P D 、P o

k h w . w w

通过每管的电流为半个余弦波,余弦波幅度I cm =电流平均值

I C0=

1I cm π

d a

2

(V CC −2V CE (sat ) ) 。π

解:(1) v A 为方波,按傅里叶级数展开,其中基波分量电压振幅V cm =

12

(V CC −2V CE (sat ) ) 2 所以 P o =V cm I cm =2

2πR L

P D =V CC I C0=

2

V CC (V CC −2V CE (sat ) ) π2R L

第 7 页 共 27 页

w . c o

22

(V −2V ) ,集电极CC CE (sat )

π2R L

V cm 2

(V CC −2V CE (sat ) ) ,= 其中平均分量

R L πR L

m

ηC =P o /P D =

V CC −2V CE (sat )

V CC

(2) P D =

2

V CC (V CC −2V CE (sat ) ) =1. 24W 2

πR L

2

(V CC −2V CE (sat ) ) 2=1. 17W 2

πR L

P o =

k h

d a

解:

w w

w .

ωt

⎧v CE =V CC −V cm cos ωt =(12−11cos ωt ) V

(1) ⎨

v V V ωt ωt =+cos =(0. 5+0. 24cos ) V BB bm ⎩BE

取ωt = 0°,30°⋅⋅⋅120°,结果如下表

0°°°°°°°

v BE 0.38 v CE 课

第 8 页 共 27 页

w .

c o

2-5 谐振功率放大器原理电路和功率管输出特性曲线如图所示,已知V CC = 12 V,V BB = 0.5 V,V cm = 11 V,V bm = 0.24 V。试在特性曲线上画出动态线。若由集电极电流i C 求得I C0 = 32 mA ,I cIm = 54 mA,试求P D 、P o 、ηC 及所需的R e 。

m

ηC =P o /P D =94. 36%

(2)

P o =

12

I cm R e 2

所以 I C0=2I c1m =276. 3mA

则 P D =V CC I C0=6. 63W , ηC =P o /P D =75. 42%

2-12 设两个谐振功率放大器具有相同的回路元件参数,它们的输出功率P o 分别为1 W和0.6 W。现若增大两放大器的V CC ,发现其中P o = 1 W放大器的输出功率增加不明显,而P o = 0.6 W放大器的输出功率增加明显,试分析其原因。若要增大P o = 1 W放大器的输出功率,试问还应同时采取什么措施(不考虑功率管的安全工作问题)?

解:P o = 1 W的放大器处于临界或欠压状态,增大V CC 时,放大器更趋于欠压状态,I c1m

略有增大。因此P o 增大不明显。(若 P o ↑ 需 V CC ↑ 同时R e ↑ 或V BB ↑)

V CC 增大,发大器趋于临界,I c1m 迅速增大,所以P o 迅P o = 0.6 W的放大器处于过压状态,

速增大。

w w

w .

k h

因为I C0

121121=i d t =i I =i cos t d t =i Cmax ,ω,ωτωτ C c1m C Cmax ππ2π∫−2ππ∫−22

π

第 9 页 共 27 页

d a

I c1m =

π

试求电源供给功率P D 、集电极效率ηC 。

解:因为V BB = V BE(on),放大器工作在甲乙类,近似作乙类,

2-8 谐振功率放大器工作在欠压区,要求输出功率P o = 5 W。己知V CC = 24 V,V BB =V BE(on),R e = 53 Ω,设集电极电流为余弦脉冲,即

⎧i Cmax cos ωt v b >0i C = ⎨

v 0≤0b ⎩

w . c o

2P o

=434mA R e

P D =V CC I C0=384mW

1

P o =V cm I cm =297mW

2

ηC =P o /P D =77. 34% R e = V cm /I c1m = 204 Ω

m

3-1 若反馈振荡器满足起振和平衡条件,则必然满足稳定条件,这种说法是否正确?为什么?

解:否。因为满足起振与平衡条件后,振荡由小到大并达到平衡。但当外界因素(T、V CC ) 变化时,平衡条件受到破坏,若不满足稳定条件,振荡器不能回到平衡状态,导致停振。

3-2 一反馈振荡器,欲减小因温度变化而使平衡条件受到破坏,从而引起振荡振幅和振荡频率的变化,应增大

振幅稳定条件:

相位稳定条件:

k h

若满足振幅稳定条件,当外界温度变化引起V i 增大时,T(ωosc ) 减小,V i 增大减缓,最终回到新的平衡点。若在新平衡点上负斜率越大,则到达新平衡点所需V i 的变化就越小,振荡振幅就越稳定。

若满足相位稳定条件,外界因素变化

阻止ω增大,

osc

最终回到新平衡点。这时,若负斜率越大,则到达新平衡点所需ωosc 的变化就越小,振荡频率就越稳定。

3-3 并联谐振回路和串联谐振回路在什么激励下(电压激励还是电流激励)才能产生负斜率的相频特性?

&的频率特性才会产生负斜率的相频特性,解:并联谐振回路在电流激励下,回路端电压V

&的频率特性才会产生负斜率的相频特回路电流I 如图(a)所示。串联谐振回路在电压激励下,

w w

性,如图(b)所示。

w .

∂ϕT (ω)

∂ωω=ωosc

第 10 页 共 27 页

d a

iA

∂T (ωosc )

∂V i

V

衡状态的过程(振幅和相位)。

解:由振荡稳定条件知:

∂ϕT (ω) ∂T (ωosc ) 和,为什么?试描述如何通过自身调节建立新平∂V i ∂ω

w .

c o

m

3-5 试判断下图所示交流通路中,哪些可能产生振荡,哪些不能产生振荡。若能产生振荡,则说明属于哪种振荡电路。

k h

d a

解:

(a) 不振。同名端接反,不满足正反馈; (b) 能振。变压器耦合反馈振荡器;

(c) 不振。不满足三点式振荡电路的组成法则;

(d) 能振。但L 2C 2回路呈感性,ωosc ω1,组成电感三点式振荡电路。

(e) 能振。计入结电容C b ′e ,组成电容三点式振荡电路。

(f) 能振。但L 1C 1回路呈容性,ωosc > ω1,L 2C 2回路呈感性,ωosc > ω2,组成电容三点式振荡电路。

3-6 试画出下图所示各振荡器的交流通路,并判断哪些电路可能产生振荡,哪些电路不能产生振荡。图中,C B 、C C 、C E 、C D 为交流旁路电容或隔直流电容,L C 为高频扼流圈,偏置电阻R B1、R B2、R G 不计。

w w

w .

第 11 页 共 27 页

w . c o

m

w w

w .

k h

(a)不振,不满足三点式振荡电路组成法则。 (b) 可振,为电容三点式振荡电路。

(c) 不振,不满足三点式振荡电路组成法则。

(d) 可振,为电容三点式振荡电路,发射结电容C b ′e 为回路电容之一。 (e) 可振,为电感三点式振荡电路。

(f) 不振,不满足三点式振荡电路组成法则。

3-7 如图所示电路为三回路振荡器的交流通路,图中f 01、f 02、f 03分别为三回路的谐振频率,试写出它们之间能满足相位平衡条件的两种关系式,并画出振荡器电路(发射极交流接

第 12 页 共 27 页

d a

解:画出的交流通路如图所示。

w . c o

m

地)。

解:(1) L 2C 2、L 1C 1若呈感性,f osc f 03,所以f 03

(2) L 2C 2、L 1C 1若呈容性,f osc > f 01、f 02,L 3C 3 呈感性,f osc f osc > f 01、f 02。

3-8 试改正如图所示振荡电路中的错误,并指出电路类型。图中C B 、C D 、C E 均为旁路电容或隔直流电容,L C 、L E 、L S 均为高频扼流圈。

k h

d a

w w

w .

解:改正后电路如图所示。

图(a)中L 改为C 1,C 1改为L 1,构成电容三点式振荡电路。 图(b)中反馈线中串接隔值电容C C ,隔断电源电压V CC 。 图(c)中去掉C E ,消除C E 对回路影响,加C B 和C C 以保证基极交流接地并隔断电源电压V CC ;

第 13 页 共 27 页

w .

c o

m

L 2改为C 1构成电容三点式振荡电路。

3-9 试运用反馈振荡原理,分析如图所示各交流通路能否振荡。

w w

w .

解:在L E 处拆环,得混合Ⅱ型等效电路如图所示。

k h

平衡条件。

图(c)负反馈,不满足正反馈条件,不振。

3-13 在下图所示的电容三点式振荡电路中,已知L = 0.5 μH ,C l = 51 pF,C 2 = 3300 pF,

C 3 =(12 ~ 250)pF ,R L = 5 kΩ,g m = 30 mS,C b ′e = 20 pF,β 足够大。Q 0 = 80,试求能够起振的频率范围,图中C B 、C C 对交流呈短路,L E 为高频扼流圈。

第 14 页 共 27 页

d a

f

i1

解:图(a)满足正反馈条件,LC 并联回路保证了相频特性负斜率,因而满足相位平衡条件。

&比V &滞后一个小于90°的相位,不满足相位图(b)不满足正反馈条件,因为反馈电压V

w . c o

m

由振幅起振条件知,g m >式中n =

1

′+ng i (1) g L n

′=由g L

11

,得R eo >4. 115k Ω +

R L R eo

则能满足起振条件的振荡频率为ω=

由图示电路知,C Σ=C 3+

当C 3 = 12pF时,C Σ = 62.23 pF,ωomax =

当C 3 = 250pF时,C Σ = 300 pF。

可见该振荡器的振荡角频率范围ωmin ~ ωmax = (102.9 ~ 179.2) × 106 rad/s, 即振荡频率范围f min ~ f max = 16.38 ~ 28.52 MHz。

别大于Q e 或小于Q e ,试用相频特性分析振荡器频率的变化。

解:振荡回路相频特性如图,可见:

w w

w .

′分′>ωo ,ϕf ′>ϕf ,Q e 3-15 一LC 振荡器,若外界因素同时引起ω0、ϕf 、Q e 变化,设ωo

k h

d a

1LC Σ

′C 1C 2

。 ′C 1+C 2

R eo

>102. 9×106rad/s。 LQ o

第 15 页 共 27 页

w . c o

=179. 2×106rad/s

m

C 11

′=C 2+C b ′e =3320pF =g m =30mS 。 =0. 015,其中C 2

′r e C 1+C 2

′>ωo 时,ωosc ′>ωosc ,且Δωosc ≈Δωo ; (1)当ωo

′′,ωosc ′′>ωosc ; (2)当ϕf ′>ϕf 时,设为ωosc

(3)当Q e 增加时,相频特性趋于陡峭,

ϕf 不变,ωosc ↓

ϕf 变化,Q e ↑→ Δ ωosc ↓,Q e ↓→ Δ ωosc ↑。

3-16 如图所示为克拉泼振荡电路,已知L = 2 μH ,C 1=1000 pF,C 2 = 4000 pF,C 3 = 70 pF,Q 0 = 100,R L = 15 kΩ,C b ′e = 10 pF,R E = 500 Ω,试估算振荡角频率ωosc 值,并求满足起振条件时的I EQmin 。设 β 很大。

ωosc ≈

w w

w .

n 2=

已知 R e0 = ωosc LQ 0 =16.9 kΩ

求得 C 1, 2=

I EQ I EQ C 1111

又 n =≈0. 2,g i =+=+≈=g m

′C 1+C 2R E r e R E V T V T

k h

1LC 3

=84. 52×106rad/s

解:振荡器的交流等效电路如图所示。由于C 1>> C3,C 2 >> C3,因而振荡角频率近似为

′=R L //R e0=7. 95k Ω,′=C 2+C b ′e =4010pF R L C 2

′C 1C 2

=800. 4pF ′C 1+C 2

C 32′′≈n 2′=50. 88Ω R L R L =0. 08,

C 3+C 1, 2

第 16 页 共 27 页

d a

w . c o

m

根据振幅起振条件,g m >

I EQ ′g ′1L

′ 即求得I EQ > 3.21mA g ′+ng ,,>L i n V T n (1−n )

3-18 试指出如图所示各振荡器电路的错误,并改正,画出正确的振荡器交流通路,指

出晶体的作用。图中C B 、C C 、C E 、C S 均为交流旁路电容或隔直流电容。

k h

d a

w w

w .

图(a)L 用C 3取代,为并联型晶体振荡器,晶体呈电感。

图(b)晶体改接到发射极,为串联型晶体振荡器,晶体呈短路元件。

3-22 试判断如图所示各RC 振荡电路中,哪些可能振荡,哪些不能振荡,并改正错误。图中,C B 、C C 、C E 、C S 对交流呈短路。

第 17 页 共 27 页

解:改正后的交流通路如图所示。

w . c o

m

解:改正后的图如图所示。

w .

k h w w

(a)为同相放大器,RC 移相网络产生180°相移,不满足相位平衡条件,因此不振。改正:将反馈线自发射极改接到基极上。

(b)中电路是反相放大器,RC 移相网络产生180°相移,满足相位平衡条件,可以振荡。

(c)中放大环节为同相放大器,RC 移相网络产生180°相移,不满足相位平衡条件,因此不振。改正:移相网络从T 2集电极改接到T 1集电极上。

(d)中放大环节为反相放大器,因为反馈环节为RC 串并联电路,相移为0°,所以放大环节应为同相放大。改正:将T 1改接成共源放大器。

3-23 图(a )所示为采用灯泡稳幅器的文氏电桥振荡器,图(b )为采用晶体二极管稳幅的文氏电桥振荡器,试指出集成运算放大器输入端的极性,并将它们改画成电桥形式的电

第 18 页 共 27 页

d a

w .

c o

m

路,指出如何实现稳幅。

解:电桥形式电路如图所示。

w w

w .

k h

(a)中灯泡是非线性器件,具有正温度系数。起振时,灯泡凉,阻值小(R t ) ,放大器增益大,便于起振。随着振荡振幅增大,温度升高,R t 增加,放大器增益相应减小,最后达到平衡。

(b)中D 1、D 2是非线性器件,其正向导通电阻阻值随信号增大而减小。起振时,D 1、D 2截止, 负反馈最弱,随着振荡加强,二极管正向电阻减小,负反馈增大,使振幅达到平衡。

4-1 如图是用频率为1 000 kHz的载波信号同时传输两路信号的频谱图。试写出它的电

压表达式,并画出相应的实现方框图。计算在单位负载上的平均功率P av 和频谱宽度BW AM 。

d a

解:(1)为二次调制的普通调幅波。

第一次调制:调制信号:F = 3 kHz

载频:f 1 = 10 kHz,f 2 = 30 kHz

第二次调制:两路已调信号叠加调制到主载频f c = 1000 kHz上。 令 Ω = 2π × 3 × 103 rad/s

ω1 = 2π × 104 rad/s ω2= 2π × 3 × 104 rad/s ωc = 2π × 106 rad/s

第一次调制:v 1(t ) = 4(1 + 0.5cosΩt )cos ω1t

第 19 页 共 27 页

w . c o

m

v 2(t ) = 2(1 + 0.4cosΩt )cos ω2t

第二次调制:v O (t ) = 5 cosωc t + [4(1 + 0.5cosΩt )cos ω1t + 2(1 + 0.4cosΩt )cos ω2t ] cosωc t = 5[1+0.8(1 + 0.5cosΩt )cos ω1t + 0.4(1 + 0.4cosΩt )cos ω2t ] cosωc t (2) 实现方框图如图所示。

V m02 = 1V,M a2 = 0.4

V m0 = 5V

12

P 0=V m 0=12. 5W

2

总平均功率P av = P 0 + P av1 + P av2 = 18.08 W 4 BW AM ○

由频谱图可知F max = 33 kHz

得 BW AM = 2F = 2(1033 −1000) = 66 kHz

4-3 试画出下列三种已调信号的波形和频谱图。已知ωc >>Ω (1) v (t ) = 5cosΩt cos ωc t (V); (2) v (t ) = 5cos(ωc +Ω) t;

(3) v (t ) = (5 + 3cosΩt ) cos ωc t 。

解:(1) 双边带调制信号(a);(2) 单边带调制信号(b);(3) 普通调幅信号(c)。

w w

w .

k h

121P 02=V m P av2=2P 02(1+M a 22) =1. 08W 02=0. 5W ;

223 主载频f c = 1000 kHz ○

第 20 页 共 27 页

d a

1212P 01=V m =2W ;P =2P (1+M a 1) =4. 5W 01av101

222 f 2 = 30 kHz ○

(3) 根据频谱图,求功率。

1 载频为10 kHz的振幅调制波平均功率 ○

V m01 = 2V,M a1 = 0.5

w . c o

m

w w

w .

式中v = V Q 十v 1+ v 2 = V Q +V 1m cos ω1t +V 2m cos ω2t 。若V 2m 很小,满足线性时变条件,则在V Q = −V 1m /2、0、V 1m 三种情况下,画出g (v 1) 波形,并求出时变增量电导g (v 1) 的表示式,分析该器件在什么条件下能实现振幅调制、解调和混频等频谱搬移功能。

解:根据伏安特性画出增量电导随v 的变化特性g (v ) 如图所示。

k h

第 21 页 共 27 页

d a

⎧g v i =⎨D

⎩0

v >0v ≤0

4-6 何谓过调幅?为何双边带调制信号和单边带调制信号均不会产生过调幅? 答:调制信号振幅大于载波信号振幅的情况称为过调幅。因为双边带和单边带调制信号已经将载波信号抑制,故均不会产生过调幅。

4-8 一非线性器件的伏安特性为

w . c o

m

k h w w

1V m

π12图中通角由cos θ==,求得θ= 3V m 2

d a

w .

1

(1)V Q =−V 1m 时,画出g (t ) 波形如图所示。

2

131g 0=g d t =g D ωD

2π∫−π33

π

2g 13n π

g n =∫πg D cos n ωt d ωt =D sin()

π−33n π

π

2g D 1

g (t ) =g D +

(2) V Q = 0时,画出g (v ) 的波形如图所示。

第 22 页 共 27 页

w . c o

1n πsin(cos n ω1t ∑n 3n =1

m

122

g (t ) =g D K 1(ω1t ) =g D (+cos ω1t −cos3ω1t +⋅⋅⋅)

2π3π

1∞2=g D [+∑(−1) n −1(2n −1) ω1t ]

2n =1(2n −1) π

k h

v c

2V T =

第 23 页 共 27 页

d a

解:由教材(4-2-14)可知

w w

其中I 0≈

又x c =

则th (

w .

令x c =

i C = iC1 − iC2 = i 0th (

V CM

,i 0 = I 0 + i Ω(t ) V T

v (t ) 11

mA ,i Ω(t ) ≈Ω=×10−3cos(2π×103t )(mA) 3R EE 3

EE −5V R EE

1

i 0=[1+10−3cos(2π×103t )](mA)

3

V cm 360mV

==13. 85>10 V T 26mV

x c 444cos ωc t ) ≈K 2(ωc t ) =cos ωc t −cos 3ωc t +cos 5ωc t −⋅⋅⋅ 2π3π5π

w .

c o

(3) V Q = V 1m ,g (t ) = gD ,如图所示。

可见,(1)、(2)中g (t ) 含有基波分量,能实现频谱搬移功能,而(3)中g (t ) 仅有直流分量,故无法实现频谱搬移功能。

为实现消除一些有害无用的组合频率分量,使输出有用信号的质量提高,在实现频谱搬移功能时,应遵循有用信号较弱,参考信号较强的原则。

调制时:v 1 = V cm cos ωc t (载波) ,v 2 = V Ωm cos Ωt (调制信号)

解调时:v 1 = V cm cos ωc t (参考信号) ,v 2 = V sm (1 + M a cos Ωt )cos ωc t (调幅信号) 混频时:v 1 = V Lm cos ωL t (本振信号) ,v 2 = V sm (1 + M a cos Ωt )cos ωc t (调幅信号)

4-9 在如图所示的差分对管调制电路中,已知v c (t ) = 360cos10π × 106t (mV ),v Ω (t ) = 5cos2π × 103t (mV ),V CC =|V EE |= 10 V,R EE =15 kΩ,晶体三极管 β 很大,V BE(on)可忽略。试用开关函数求i C =(i C1 − i C2)值。

m

所以

i C =i 0th (

x c 1

cos ωc t ) ≈[1+10−3cos(2π×103t )]K 2(ωc t )

32

=[1+10−3cos(2π×103t )][0. 42cos(10π×106t ) −0. 14cos(30π×106t ) +0. 084cos(50π×106t ) −⋅⋅⋅](mA )

4-11一双差分对平衡调制器如图所示,其单端输出电流

I i −i I qv v qv i I =0+56th 1≈0+2th 1

222kT 2R E 2kT

d a

解:(1) 混频:v 1(t ) = vL (t ) =V Lm cos ωL t ,v 2(t ) = vS (t ) = V sm cos ωc t ,当V Lm > 260 mV,V sm

mV 工作在开关状态时,产生的组合频率分量有ωL ± ωc ,3ωL ± ωc ,⋅⋅⋅,(2n +1)ωL ± ωc ,输出采用中心频率为 ωI 的带通滤波器。

(2) 双边带调制:v 1(t ) = v c (t ) = V cm cos ωc t ,v 2(t ) = vΩ(t ) = V Ωm (t )cos Ωt 。

工作在开关状态时,产生的组合频率分量有ωc ± Ω,3ωc ± Ω,⋅⋅⋅,(2n +1)ωc ± Ω。输出采用中心频率为 ωc ,BW 0.7 > 2F 的带通滤波器。

(3) 双边带调制波解调:v 1(t ) = v r (t ) = V rm cos ωc t ,v 2(t ) = vS (t ) = V m0cos Ωt cosωc t 。开关工作时,产生的组合频率分量有Ω,2ωc ± Ω,4ωc ± Ω,⋅⋅⋅,2n ωc ± Ω。输出采用低通滤波器,BW 0.7 > 2F 。

4-16 采用双平衡混频组件作为振幅调制器,如图所示。图中v c (t ) = V cm cos ωc t ,v Ω(t ) = V Ωm cos Ωt 。各二极管正向导通电阻为R D ,且工作在受v C (t ) 控制的开关状态。设R L >>R D ,试求输出电压v O (t ) 表达式。

解:作混频器,且v C >> v Ω,各二极管均工作在受v C 控制的开关状态。 当 v C > 0,D 1、D 2导通,D 3、D 4截止 当 v C 0时,等效电路,i I = i 1 − i 2 回路方程为:

w w

w .

k h

⎧v Ω−v C +i 1R D +(i 1−i 2) R L =0⎨

⎩−v Ω−i I R L +i 2R D −v C =0

第 24 页 共 27 页

w . c o

①②

1 − ○2 ○

m

试分析为实现下列功能(不失真),两输人端各自应加什么信号电压?输出端电流包含哪些

频率分量,输出滤波器的要求是什么?

(1)混频(取ωI =ωL − ωC );(2)双边带调制;(3)双边带调制波解调。

2( i1 − i 2) R L + 2 vΩ + ( i1 − i 2) R D = 0

2v Ω

i I =i 1−i 2=−

2R L +R D 考虑v C 作为开关函数K 1(ωc t ) 所以 i I =−

2v Ω(t )

K 1(ωc t )

2R L +R D

(2) 同理可求v C > R D ∴v O (t ) =−

2v Ω(t ) 2v Ω(t )

[K 1(ωc t ) −K 1(ωc t −π)]=−K 2(ωc t )

2R L +R D 2R L +R D

23

= I 0+av be +bv be +cv be 。设还有一干扰信号v M =V Mm cos (2π×3.5×105t ),作用于混频器的

234

+cv be +dv be ,求其中交叉调制失真的振幅。(3)若改用场效应转移特性为i c =I 0+av be +bv be

w w

2

= 300 kHz,表明频率为f M 的干扰信号可在混频器输出,它由静态转移特性三次方项中3cv L v M 项产生。

42=d (v S +v L +v M ) 4中产生6d (v S +v L ) 2v M 分量,而(2) 静态特性四次方项dv be

222212

产生中频 ωI 分6d (v S +v L ) 2v M =6d (v S +2v S v L +v L ) V Mm (1+cos 2ωM t ) 中分量6v S v L V Mm

2

2

量,其幅值为3dV sm V Lm V Mm ,包含了干扰信号包络变化造成的交叉失真。

(3)由于干扰频率只能通过器件特性的三次方以上项才能产生中频频率,所以工作在平方律特性曲线内,无干扰信号的影响。

4-24 混频器中晶体三极管在静态工作点上展开的转移特性由下列幂级数表示:i C = I 0+

234

av be +bv be +cv be +dv be 。已知混频器的本振频率为f L = 23 MHz,中频频率f I = f L − f c = 3 MHz。

若在混频器输人端同时作用f M1 = 19.6 MHz和f M2=19.2 MHz的干扰信号。试问在混频器输出端是否会有中频信号输出?它是通过转移特性的几次项产生的?

第 25 页 共 27 页

w .

管,器件工作在平方律特性的范围内,试分析干扰信号的影响。

q p ±1

解:(1) f M = 350 kHz,f c = 300 kHz,由f c =f M −p = 1,q = 2时,2f M − 2f 2 f I 得知,

p p

k h

输人端。试问:(1)干扰信号v M 通过什么寄生通道变成混频器输出端的中频电压?(2)若

d a

4-23 晶体三极管混频器的输出中频频率为f I = 200 kHz,本振频率为f L = 500 kHz,输人

信号频率为f c = 300 kHz。晶体三极管的静态转移特性在静态偏置电压上的幂级数展开式为i C

2R L

v Ω(t ) K 2(ωc t ) ≈−v Ω(t ) K 2(ωc t )

2R L +R D

w . c o

m

2v Ω(t )

K 1(ωc t −π)

2R L +R D

1时,f p, q, r, s =f L −(2f M1−f M2) =3MHz , 产生中频信号输出。可见它是由转移特性四次方

42项v be =4(v L +v M1+v M2) 4中12v L v M1v M2分量产生的,被称为互调失真,其振幅为12dV L m V Mm1V Mm2。 2

4-27 如图所示为发送两路语言信号的单边带发射机,试画出(A ~F )各点的频谱图,图中,频率合成器提供各载波频率信号。

解:A~F各点频谱图如图所示。

w .

4-30 包络检波电路如图所示,二极管正向电阻R D = 100 Ω,F =(100 ~ 5000)Hz 。图(a )中,M amax = 0.8;图(b )中M a = 0.3。试求图(a )中电路不产生负峰切割失真和惰性失真的C 和R i2值。图(b )中当可变电阻R 2的接触点在中心位置时,是否会产生负峰切割失真?

解:(1) 图(a)中,已知R L = R L1 + RL2 = 5 kΩ,Ωmax = 2π × 5000 rad/s,M amax = 0.8,根据不产生惰性失真条件,得

=4775pF

R L Ωmax M a max

(2) 根据不产生负峰切割失真条件得 Z L (Ω) ≥ M a Z L (0) = M amax R L = 4 kΩ 因为Z L (Ω) = R L1 + R L2 // R i2,

Z (Ω) R L 1+R L 2//R i 21+4//R i 2

=,R i 2>12k Ω =M amax = 0.8

Z L (0) R L 1+R L 25

(3) R L 在中间位置时

R R

Z L (Ω) =R 1+2//R i =1211. 5Ω,Z L (0) =R 1+2=2860Ω

22

w w

所以

Z L (Ω)

=0. 42>0. 3 故不产生负峰切割失真。 Z L (0)

第 26 页 共 27 页

k h

C ≤

d a

−M a 2max

w . c o

m

5-1 一已调波v (t ) = V m cos(ωc + A ω1t ) t ,试求它的Δϕ(t ) 、Δω (t ) 的表示式。如果它是调频波或调相波,试问,它们相应的调制电压各为什么?

d Δϕ(t )

解:Δϕ(t ) = A ω1t 2,Δω(t ) = =2A ω1t 。

d t

若为调频波,则由于瞬时频率变化Δω (t ) 与调制信号成正比,即

1

2A ω1t Δω (t ) = k f v Ω(t ) = 2A ω1t ,所以调制电压v Ω(t ) =k f

若为调相波,则由于瞬时相位变化Δϕ(t ) 与调制信号成正比,即

1

Δϕ(t ) = k p v Ω(t ) = A ω1t 2,所以调制电压v Ω(t ) =A ω1t 2

k p

w w

w .

第 27 页 共 27 页

k h

(2) 对应两种调制信号调频波FM 和调相波PM 的Δω (t ) 和Δϕ(t ) 分别如图(a)、(b)所示。

d a

w . c o

5-2 已知载波信号v C (t ) = V cm cos ωc t ,调制信号为周期性方波和三角波,分别如图(a )

(1)调幅波,调频波;(2)调频波和调相波的瞬时角频率和(b )所示。试画出下列波形:

。 偏移Δω(t ) 。瞬时相位偏移Δϕ(t ) (坐标对齐)

解:(1) 对应两种调制信号画出调幅波和调频波的波形分别如图(a)、(b)所示。

m


© 2024 实用范文网 | 联系我们: webmaster# 6400.net.cn